Difference between revisions of "2007 AIME II Problems/Problem 9"

m (Solution 2: typo)
(Solution: add own incomplete solution)
Line 8: Line 8:
 
Several [[Pythagorean triple]]s exist amongst the numbers given. <math>BE = DF = \sqrt{63^2 + 84^2} = 21\sqrt{3^2 + 4^2} = 105</math>. Also, the length of <math>EF = \sqrt{63^2 + (448 - 2\cdot84)^2} = 7\sqrt{9^2 + 40^2} = 287</math>.
 
Several [[Pythagorean triple]]s exist amongst the numbers given. <math>BE = DF = \sqrt{63^2 + 84^2} = 21\sqrt{3^2 + 4^2} = 105</math>. Also, the length of <math>EF = \sqrt{63^2 + (448 - 2\cdot84)^2} = 7\sqrt{9^2 + 40^2} = 287</math>.
  
Use the [[Two Tangent theorem]] on <math>\triangle BEF</math>. Since both circles are inscribed in congruent triangles, they are congruent; therefore, <math>EP = FQ = \frac{287 - PQ}{2}</math>. By the Two Tangent theorem, note that <math>EP = EX = \frac{287 - PQ}{2}</math>, making <math>\displaystyle BX = 105 - EX = 105 - \left[\frac{287 - PQ}{2}\right]</math>. Also, <math>\displaystyle BX = BY</math>. <math>FY = 364 - BY = 364 - \left[105 - \left[\frac{287 - PQ}{2}\right]\right]</math>.  
+
Use the [[Two Tangent theorem]] on <math>\triangle BEF</math>. Since both circles are inscribed in congruent triangles, they are congruent; therefore, <math>EP = FQ = \frac{287 - PQ}{2}</math>. By the Two Tangent theorem, note that <math>EP = EX = \frac{287 - PQ}{2}</math>, making <math>BX = 105 - EX = 105 - \left[\frac{287 - PQ}{2}\right]</math>. Also, <math>BX = BY</math>. <math>FY = 364 - BY = 364 - \left[105 - \left[\frac{287 - PQ}{2}\right]\right]</math>.  
  
Finally, <math>FP = FY = 364 - \left[105 - \left[\frac{287 - PQ}{2}\right]\right] = \frac{805 - PQ}{2}</math>. Also, <math>FP = FQ + PQ = \frac{287 - PQ}{2} + PQ</math>. Equating, we see that <math>\frac{805 - PQ}{2} = \frac{287 + PQ}{2}</math>, so <math>\displaystyle PQ = 259</math>.
+
Finally, <math>FP = FY = 364 - \left[105 - \left[\frac{287 - PQ}{2}\right]\right] = \frac{805 - PQ}{2}</math>. Also, <math>FP = FQ + PQ = \frac{287 - PQ}{2} + PQ</math>. Equating, we see that <math>\frac{805 - PQ}{2} = \frac{287 + PQ}{2}</math>, so <math>PQ = 259</math>.
  
 
=== Solution 2 ===
 
=== Solution 2 ===
By the [[Two Tangent theorem]], we have that <math>\displaystyle FY = PQ + QF</math>. Solve for <math>\displaystyle PQ = FY - QF</math>. Also, <math>\displaystyle QF = EP = EX</math>, so <math>\displaystyle PQ = FY - EX</math>. Since <math>\displaystyle BX = BY</math>, this can become <math>\displaystyle PQ = FY - EX + (BY - BX)</math><math> = \left(FY + BY\right) - \left(EX + EY\right) = FB - EB</math>. Substituting in their values, the answer is <math>364 - 105 = 259</math>.
+
By the [[Two Tangent theorem]], we have that <math>FY = PQ + QF</math>. Solve for <math>PQ = FY - QF</math>. Also, <math>QF = EP = EX</math>, so <math>PQ = FY - EX</math>. Since <math>BX = BY</math>, this can become <math>PQ = FY - EX + (BY - BX)</math><math> = \left(FY + BY\right) - \left(EX + EY\right) = FB - EB</math>. Substituting in their values, the answer is <math>364 - 105 = 259</math>.
 +
 
 +
===Solution 3===
 +
 
 +
Call the incenter of <math>\triangle BEF</math> <math>O_1</math> and the incenter of that, uh, other triangle <math>O_2</math>. Draw triangles <math>\triangle O_1PQ,\triangle PQO_2</math>.
 +
 
 +
Drawing <math>BE</math>, We find that <math>BE = \sqrt {63^2 + 84^2} = 105</math>. Applying the same thing for <math>F</math>, we find that <math>FD = 105</math> as well. Draw a line through <math>E,F</math> parallel to the sides of the rectangle, to intersect the opposite side at <math>E_1,F_1</math> respectively. Drawing <math>\triangle EE_1F</math> and <math>FF_1E</math>, we can find that <math>EF = \sqrt {63^2 + 280^2} = 287</math>. We then use Heron's formula to get:
 +
\[
 +
[BEF] = [DEF] = 11 466
 +
\]
 +
.
 +
 
 +
So the inradius of the triangle-type things is <math>\frac {637}{21}</math>.
 +
 
 +
Now, we just have to find <math>O_1Q = O_2P</math>, which can be done with simple subtraction, and then we can use the [[Pythagorean Theorem]] to find <math>PQ</math>.
 +
 
 +
{{incomplete|solution}}
  
 
== See also ==
 
== See also ==

Revision as of 16:15, 8 December 2007

Problem

Rectangle $ABCD$ is given with $AB=63$ and $BC=448.$ Points $E$ and $F$ lie on $AD$ and $BC$ respectively, such that $AE=CF=84.$ The inscribed circle of triangle $BEF$ is tangent to $EF$ at point $P,$ and the inscribed circle of triangle $DEF$ is tangent to $EF$ at point $Q.$ Find $PQ.$

Solution

2007 AIME II-9.png

Solution 1

Several Pythagorean triples exist amongst the numbers given. $BE = DF = \sqrt{63^2 + 84^2} = 21\sqrt{3^2 + 4^2} = 105$. Also, the length of $EF = \sqrt{63^2 + (448 - 2\cdot84)^2} = 7\sqrt{9^2 + 40^2} = 287$.

Use the Two Tangent theorem on $\triangle BEF$. Since both circles are inscribed in congruent triangles, they are congruent; therefore, $EP = FQ = \frac{287 - PQ}{2}$. By the Two Tangent theorem, note that $EP = EX = \frac{287 - PQ}{2}$, making $BX = 105 - EX = 105 - \left[\frac{287 - PQ}{2}\right]$. Also, $BX = BY$. $FY = 364 - BY = 364 - \left[105 - \left[\frac{287 - PQ}{2}\right]\right]$.

Finally, $FP = FY = 364 - \left[105 - \left[\frac{287 - PQ}{2}\right]\right] = \frac{805 - PQ}{2}$. Also, $FP = FQ + PQ = \frac{287 - PQ}{2} + PQ$. Equating, we see that $\frac{805 - PQ}{2} = \frac{287 + PQ}{2}$, so $PQ = 259$.

Solution 2

By the Two Tangent theorem, we have that $FY = PQ + QF$. Solve for $PQ = FY - QF$. Also, $QF = EP = EX$, so $PQ = FY - EX$. Since $BX = BY$, this can become $PQ = FY - EX + (BY - BX)$$= \left(FY + BY\right) - \left(EX + EY\right) = FB - EB$. Substituting in their values, the answer is $364 - 105 = 259$.

Solution 3

Call the incenter of $\triangle BEF$ $O_1$ and the incenter of that, uh, other triangle $O_2$. Draw triangles $\triangle O_1PQ,\triangle PQO_2$.

Drawing $BE$, We find that $BE = \sqrt {63^2 + 84^2} = 105$. Applying the same thing for $F$, we find that $FD = 105$ as well. Draw a line through $E,F$ parallel to the sides of the rectangle, to intersect the opposite side at $E_1,F_1$ respectively. Drawing $\triangle EE_1F$ and $FF_1E$, we can find that $EF = \sqrt {63^2 + 280^2} = 287$. We then use Heron's formula to get: \[ [BEF] = [DEF] = 11 466 \] .

So the inradius of the triangle-type things is $\frac {637}{21}$.

Now, we just have to find $O_1Q = O_2P$, which can be done with simple subtraction, and then we can use the Pythagorean Theorem to find $PQ$.

Template:Incomplete

See also

2007 AIME II (ProblemsAnswer KeyResources)
Preceded by
Problem 8
Followed by
Problem 10
1 2 3 4 5 6 7 8 9 10 11 12 13 14 15
All AIME Problems and Solutions